Prove Vector Projections Perpendicular in R3

  • Thread starter Thread starter tony873004
  • Start date Start date
  • Tags Tags
    Projections Vector
Click For Summary
To prove that the vector w minus its projection onto vector v is perpendicular to v, start with the equation (w - proj_v w) · v = 0. The projection of w onto v is given by proj_v w = (v · w / |v|^2)v. By substituting this into the equation, it simplifies to showing that v · (w - (v · w / |v|^2)v) equals zero. This confirms that the difference vector is indeed orthogonal to v. The discussion emphasizes the importance of simplifying the dot product to demonstrate the perpendicularity clearly.
tony873004
Science Advisor
Gold Member
Messages
1,753
Reaction score
143
Let \overrightarrow v and \overrightarrow w be vectors in R3. Prove that \overrightarrow w - {\rm{proj}}_{\overrightarrow v } \overrightarrow w is perpendicular to \overrightarrow v .

Here's my attempt:
\begin{array}{l}<br /> \left( {\overrightarrow w - {\rm{proj}}_{\overrightarrow v } \overrightarrow w } \right) \cdot \overrightarrow v \mathop = \limits^? 0 \\ <br /> \\ <br /> {\rm{proj}}_{\overrightarrow v } \overrightarrow w = \frac{{\overrightarrow v \cdot \overrightarrow w }}{{\left| {\overrightarrow v } \right|^2 }}\overrightarrow v \\ <br /> \\ <br /> \overrightarrow v \cdot \overrightarrow w = v_1 w_1 + v_2 w_2 + v_3 w_3 \\ <br /> \\ <br /> \left| {\overrightarrow v } \right| = \sqrt {v_1^2 + v_2^2 + v_3^2 } \\ <br /> \left| {\overrightarrow v } \right|^2 = v_1^2 + v_2^2 + v_3^2 \\ <br /> \\ <br /> \frac{{\overrightarrow v \cdot \overrightarrow w }}{{\left| {\overrightarrow v } \right|^2 }} = \frac{{v_1 w_1 + v_2 w_2 + v_3 w_3 }}{{v_1^2 + v_2^2 + v_3^2 }} = \frac{{v_1 w_1 }}{{v_1^2 + v_2^2 + v_3^2 }} + \frac{{v_2 w_2 }}{{v_1^2 + v_2^2 + v_3^2 }} + \frac{{v_3 w_3 }}{{v_1^2 + v_2^2 + v_3^2 }} \\ <br /> \\ <br /> \frac{{\overrightarrow v \cdot \overrightarrow w }}{{\left| {\overrightarrow v } \right|^2 }}\overrightarrow v = \left( {\frac{{v_1 w_1 }}{{v_1^2 + v_2^2 + v_3^2 }} + \frac{{v_2 w_2 }}{{v_1^2 + v_2^2 + v_3^2 }} + \frac{{v_3 w_3 }}{{v_1^2 + v_2^2 + v_3^2 }}} \right)\left\langle {v_1 ,\,v_2 ,\,v_3 } \right\rangle \\ <br /> \\ <br /> \,\,\,\,\,\,\,\,\,\,\,\,\,\, = \left( {\frac{{v_1 w_1 }}{{v_1^2 + v_2^2 + v_3^2 }} + \frac{{v_2 w_2 }}{{v_1^2 + v_2^2 + v_3^2 }} + \frac{{v_3 w_3 }}{{v_1^2 + v_2^2 + v_3^2 }}} \right)\left\langle {v_1 ,\,v_2 ,\,v_3 } \right\rangle \\ <br /> \end{array}

Things are starting to get real ugly. Am I missing an easier way?
 
Physics news on Phys.org
Yes, you are. You want to show v.(w-(v.w)*v/(v.v))=0. Just multiply the outer dot product through.
 
Question: A clock's minute hand has length 4 and its hour hand has length 3. What is the distance between the tips at the moment when it is increasing most rapidly?(Putnam Exam Question) Answer: Making assumption that both the hands moves at constant angular velocities, the answer is ## \sqrt{7} .## But don't you think this assumption is somewhat doubtful and wrong?

Similar threads

  • · Replies 3 ·
Replies
3
Views
1K
Replies
5
Views
2K
  • · Replies 3 ·
Replies
3
Views
3K
  • · Replies 4 ·
Replies
4
Views
2K
  • · Replies 2 ·
Replies
2
Views
2K
  • · Replies 9 ·
Replies
9
Views
3K
  • · Replies 3 ·
Replies
3
Views
1K
  • · Replies 6 ·
Replies
6
Views
4K
  • · Replies 2 ·
Replies
2
Views
2K
  • · Replies 4 ·
Replies
4
Views
2K